What Are the Kernel and Image of the Linear Map T?

Click For Summary
The discussion revolves around finding the kernel and image of the linear map T defined by T(a,b) = (a+b, 2a+2b). The kernel consists of all vectors (a,b) such that T(a,b) equals (0,0), which requires solving the equations a + b = 0 and 2a + 2b = 0. The image of T is the set of all output vectors produced by T, which can be visualized by examining how T transforms R^2. Additionally, the standard basis for R^2 refers to the vectors (1,0) and (0,1), which are used to express the matrix representation of T. Understanding these concepts is essential for solving the homework problem effectively.
mrroboto
Messages
35
Reaction score
0

Homework Statement



Let T (element of L(R^2,R^2) ) be the linear map T(a,b) = (a+b, 2a +2b)

A) What is the kernal of T

B) What is the image of T

C) Give the matrix for T in the standard basis for R^2

Homework Equations



Kernal of T = {v element of V st T(v) = 0}
Image of T = {w element of W st T(v) = w}

I'm not sure about the matrix


The Attempt at a Solution



I'm really not sure where to go with this. In this case, there are two variable (a,b) instead of 1 variable (v), so I don't know how either the kernal or the image work.

I don't know what standard basis means, and I can't find it in my notes.

Can someone help me?
 
Physics news on Phys.org
Well T is an element of the dual space and it is a map. you want to find the set of points (a,b) such that T(a,b) = (0,0)

As for the image of T try to visualise what the ap is doing to R2

The standard basis is what we normally use as a set of basis vectors, which ill leave you to find out (its pretty obvious youll get it)
 
The key for the standard basis is R^2. It's different for all R^n, so focus on the two
 
Last edited:
Question: A clock's minute hand has length 4 and its hour hand has length 3. What is the distance between the tips at the moment when it is increasing most rapidly?(Putnam Exam Question) Answer: Making assumption that both the hands moves at constant angular velocities, the answer is ## \sqrt{7} .## But don't you think this assumption is somewhat doubtful and wrong?

Similar threads

  • · Replies 1 ·
Replies
1
Views
2K
  • · Replies 1 ·
Replies
1
Views
1K
Replies
1
Views
1K
  • · Replies 24 ·
Replies
24
Views
4K
Replies
15
Views
2K
Replies
8
Views
2K
Replies
9
Views
2K
  • · Replies 5 ·
Replies
5
Views
2K
  • · Replies 4 ·
Replies
4
Views
2K
Replies
5
Views
2K